\begin{proof}Poiché $f_1$ ed $f_2$ sono derivabili in $\xbar$, vale
\begin{proof}Poiché $f_1$ ed $f_2$ sono derivabili in $\xbar$, vale
che:
che:
\[ f_1(\xbar+ h)= f_1(\xbar)+ f_1'(\xbar) h + o(h), \qquad f_2(\xbar+ h)= f_2(\xbar)+ f_2'(\xbar) h + o(h). \]
\[ f_1(\xbar+ h)= f_1(\xbar)+ f_1'(\xbar) h + o(h), \qquad f_2(\xbar+ h)= f_2(\xbar)+ f_2'(\xbar) h + o(h). \]
\begin{enumerate}[(i)]
\begin{enumerate}[(i)]
@ -156,12 +153,11 @@
\begin{enumerate}[(i)]
\begin{enumerate}[(i)]
\item$\ybar$ è un punto di accumulazione di $Y$,
\item$\ybar$ è un punto di accumulazione di $Y$,
\item$g$ è derivabile in $\ybar$,
\item$g$ è derivabile in $\ybar$ e $g'(\ybar)=\frac{1}{f'(\xbar)}$.
\item$g'(\ybar)=\frac{1}{f'(\xbar)}$.
\end{enumerate}
\end{enumerate}
\end{proposition}
\end{proposition}
\begin{proof}\nl
\begin{proof} Si dimostrano i due risultati separatamente.
\begin{enumerate}[(i)]
\begin{enumerate}[(i)]
\item Poichè $f$ è derivabile in $\xbar$, $f$ è continua
\item Poichè $f$ è derivabile in $\xbar$, $f$ è continua
in $\xbar$. Quindi per ogni intorno $I$ di $\ybar$, esiste
in $\xbar$. Quindi per ogni intorno $I$ di $\ybar$, esiste
@ -169,7 +165,7 @@
è mai vuoto, dacché, essendo $f$ derivabile in $\xbar$, $\xbar$ è un punto di accumulazione di $X$. Quindi $J$ contiene in particolare un immagine di $f$ in esso, e quindi un punto di $Y$;
è mai vuoto, dacché, essendo $f$ derivabile in $\xbar$, $\xbar$ è un punto di accumulazione di $X$. Quindi $J$ contiene in particolare un immagine di $f$ in esso, e quindi un punto di $Y$;
inoltre, tale punto è diverso da $\ybar$ dal momento che $f$ è
inoltre, tale punto è diverso da $\ybar$ dal momento che $f$ è
iniettiva, essendo bigettiva. Quindi $\ybar$ è un punto di accumulazione.
iniettiva, essendo bigettiva. Quindi $\ybar$ è un punto di accumulazione.
\iteme \!(iii) Poiché $f$ è derivabile in $g(\ybar)$,
\item Poiché $f$ è derivabile in $g(\ybar)$,
$\ybar+ h = f(g(\ybar+ h))= f(g(\ybar)+(\underbrace{g(\ybar+ h)- g(\ybar)}_k))=\ybar+ f'(\xbar) k +
$\ybar+ h = f(g(\ybar+ h))= f(g(\ybar)+(\underbrace{g(\ybar+ h)- g(\ybar)}_k))=\ybar+ f'(\xbar) k +
o(k)$, ossia vale che:
o(k)$, ossia vale che:
@ -188,6 +184,8 @@
\[ f(x)=\system{x &\se x \geq0, \\-(x+2)&\se-2 < x \leq-1.}\]
\[ f(x)=\system{x &\se x \geq0, \\-(x+2)&\se-2 < x \leq-1.}\]
\vskip 0.05in
dove $f'(0)=1$, $f$ è invertibile, ma la derivata di $g$ in $0$ non
dove $f'(0)=1$, $f$ è invertibile, ma la derivata di $g$ in $0$ non
esiste ($D_+ g(0)=1$, ma $D_- g(0)=+\infty$).
esiste ($D_+ g(0)=1$, ma $D_- g(0)=+\infty$).
\end{example}
\end{example}
@ -198,10 +196,34 @@
$f'(\xbar)=0$.
$f'(\xbar)=0$.
\end{theorem}
\end{theorem}
\begin{example}
\begin{proof}
Dimostrare che la derivata sinistra è negativa, e che quella
Poiché $I$ è un intervallo e $\xbar$ è interno a $I$, $\xbar$ è sia punto di accumulazione
destra è positiva nei casi che hai capito.
sinistro che punto di accumulazione destro di $I$. Dal momento che $f$ è derivabile
\end{example}
in $\xbar$, esistono sia la derivata destra che la derivata sinistra in $\xbar$. \\
Si assuma che $\xbar$ è un punto di massimo locale (altrimenti è sufficiente considerare $g =-f$).
Allora esiste un intorno $J$ di $\xbar$ tale per cui $x \in J \implies f(x)- f(\xbar)\leq0$. Sia dunque
$J_+$ l'intorno destro relativo a $J$, e sia $J_-$ quello sinistro. \\
Poiché $\xbar=\inf J_+$, esiste una successione $\{x_n\}\subseteq J_+\setminus\{\xbar\}$ tale
per cui $x_n \tendston\xbar$. Dal momento che allora $f$ è derivabile in $\xbar$, $f$ è anche
continua in $\xbar$, e quindi si ricava che $f(x_n)\tendston f(\xbar)$. Si osserva dunque
che $f(x_n)- f(\xbar)\leq0$ e $x_n -\xbar > 0\implies\frac{f(x_n)- f(\xbar)}{x_n -\xbar}\leq0$,
da cui, per il teorema della permanenza del segno, si ricava che $L_+=\lim_{n \to\infty}\frac{f(x_n)- f(\xbar)}{x_n -\xbar}\leq0$. \\
Allora, dal momento che $f$ è derivabile in $\xbar$ e che la derivata destra
deve coincidere con la derivata classica, $f'(\xbar)=
Analogamente si ricava che $f'(\xbar)\geq0$, e quindi che $f'(\xbar)$ è necessariamente pari a zero, da cui la
tesi.
\end{proof}
\begin{remark}\nl
\li Si può facilmente generalizzare il teorema di Fermat assumendo ipotesi più deboli. Sia infatti $x_M$ un punto di massimo locale e sia $f$ continua in $x_M$,
allora, qualora esistano, $D_+ f(x_M)\leq0$ e $D_- f(x_M)\geq0$. Analogamente
si estende la proposizione a $x_m$ punto di minimo locale.
\end{remark}
\begin{theorem} (di Rolle)
\begin{theorem} (di Rolle)
Sia $I =[a, b]\subset\RR$ e sia $f : I \to\RR$ tale che
Sia $I =[a, b]\subset\RR$ e sia $f : I \to\RR$ tale che
@ -212,11 +234,10 @@
\begin{proof}
\begin{proof}
Per il teorema di Weierstrass $f$ ammette un punto di massimo $M$ e uno di minimo $m$ in $I$. Se $f(a)= M$ e $f(b)= m$ o viceversa, la
Per il teorema di Weierstrass $f$ ammette un punto di massimo $M$ e uno di minimo $m$ in $I$. Se $f(a)= M$ e $f(b)= m$ o viceversa, la
funzione $f$ è costante in $I$, e quindi per ogni punto in $(a, b)$
funzione $f$ è costante in $I$, e quindi per ogni punto in $(a, b)$
la derivata è nulla, dacché $f$ è sempre derivabile. Altrimenti,
la derivata è nulla. Altrimenti,
sicuramente uno tra il punto di massimo e quello di minimo appartiene
sicuramente uno tra il punto di massimo e quello di minimo appartiene
a $(a, b)$. Senza perdita di generalità, si assuma che $\exists x_M \in(a, b)$ tale che $f(x_M)= M$: per
a $(a, b)$. Sia $\xbar$ tale punto. Allora, per il teorema di Fermat, $f'(\xbar)=0$, da cui
il teorema di Fermat $f'(x_M)=0$. Analogamente per il caso in cui
la tesi.
$\exists x_m \in(a, b)$ tale che $f(x_m)= m$, da cui la tesi.
\end{proof}
\end{proof}
\begin{theorem} (di Cauchy)
\begin{theorem} (di Cauchy)
@ -238,12 +259,12 @@
\begin{theorem} (di Lagrange)
\begin{theorem} (di Lagrange)
Sia $I =[a, b]\subset\RR$ e sia $f: I \to\RR$ tale che $f$
Sia $I =[a, b]\subset\RR$ e sia $f: I \to\RR$ tale che $f$
sia continua su $I$ e che $f$ sia derivabile in $(a, b)$. Allora
sia continua su $I$ e che $f$ sia derivabile in $(a, b)$. Allora
$\exists\xbar\in(a, b)$ tale che $f'(\xbar)=\frac{f(b)- f(a)}{b-a}$, ossia la cui retta tangente è parallela alla secante
$\exists\xbar\in(a, b)$ tale che $f'(\xbar)=\frac{f(b)- f(a)}{b-a}$, ossia tale per cui la retta tangente a $f$ in $\xbar$ è parallela alla secante
che passa per $(a, f(a))$ e $(b, f(b))$.
che passa per $(a, f(a))$ e $(b, f(b))$.
\end{theorem}
\end{theorem}
\begin{proof}
\begin{proof}
Si consideri $g(x)= x$,$g$ è continua in $[a, b]$ e derivabile
Si consideri $g(x)= x$.$g$ è continua in $[a, b]$ e derivabile
in $(a, b)$, con derivata sempre non nulla in tale intervallo.
in $(a, b)$, con derivata sempre non nulla in tale intervallo.
Allora, per il teorema di Cauchy, $\exists\xbar\in(a, b)\mid
Allora, per il teorema di Cauchy, $\exists\xbar\in(a, b)\mid
f'(\xbar) = \frac{f(b)-f(a)}{b-a}$, da cui la tesi.
f'(\xbar) = \frac{f(b)-f(a)}{b-a}$, da cui la tesi.
@ -263,13 +284,13 @@
Si considerino $c < d \in I$. Allora, per il teorema di Lagrange,
Si considerino $c < d \in I$. Allora, per il teorema di Lagrange,
f(d) \geq f(c)$, e quindi$f$ è crescente in $I$, da cui la tesi.
\end{proof}
\end{proof}
\begin{remark}\nl
\begin{remark}\nl
\li L'interpretazione geometrica del teorema di Cauchy, rispetto
\li L'interpretazione geometrica del teorema di Cauchy, rispetto
a quella di Lagrange, è leggermente più complicata. Si consideri
a quella di Lagrange, è leggermente più complicata. Si consideri
la curva $\gamma : \RR\to\RR^2$ tale che
la curva continua $\gamma : \RR\to\RR^2$ tale che
$\gamma(t)=(g(t), f(t))$. Si osserva che il coefficiente della
$\gamma(t)=(g(t), f(t))$. Si osserva che il coefficiente della
retta tangente in $\xbar$ per $\gamma$ è dato da $\lim_{h \to0}\frac{f(\xbar+ h)- f(\xbar)}{g(\xbar+ h)- g(\xbar)}$, che,
retta tangente in $\xbar$ per $\gamma$ è dato da $\lim_{h \to0}\frac{f(\xbar+ h)- f(\xbar)}{g(\xbar+ h)- g(\xbar)}$, che,
sotto le ipotesi del teorema di Cauchy, può essere riscritto
sotto le ipotesi del teorema di Cauchy, può essere riscritto
@ -280,43 +301,68 @@
\end{remark}
\end{remark}
\begin{exercise}
\begin{exercise}
Dare un esempio di una funzione $f : \RR\to\RR$ crescente e
Si descriva un insieme $X$ tale che i suoi unici punti di accumulazione
discontinua $\forall x \in\ZZ$.
siano $\pm1$.
\end{exercise}
\end{exercise}
\begin{solution}
\begin{solution}
Si consideri $f(x)=\lfloor x \rfloor$.
Si consideri $X =\{1+\frac{1}{n}\}\cup\{-1-\frac{1}{n}\}$, al variare di $n \in\NN$. Sia
$J =[1-\eps, 1+\eps]$ un intorno di $1$. Allora $1+\frac{1}{n}\in J$ per $n > \frac{1}{\eps}$,
da cui si ricava che $1$ è un punto di accumulazione di $X$; analogamente si verifica che $-1$ è
un punto di accumulazione di $X$. Si consideri adesso l'intorno $J =\left[1+\frac{1}{n}-\frac{1}{2}\left(\frac{1}{n}-\frac{1}{n+1}\right), 1+\frac{1}{n}+\frac{1}{2}\left(\frac{1}{n}-\frac{1}{n+1}\right)\right]$. Si verifica che nessun punto di $X$, oltre $1+\frac{1}{n}$ appartiene a $J$, e quindi
$1+\frac{1}{n}$ non è punto di accumulazione di $X$. Analogamente non lo è alcun numero della forma
$-1-\frac{1}{n}$.
\end{solution}
\end{solution}
\begin{exercise}
\begin{exercise}
Si descriva un insieme $X$ tale che i suoi punti di accumulazione
Sia $f : X \to\RRbar$ continua in $\xbar$ e sia $a < f(\xbar)$.
sono $\{\pm1\}$.
Allora esiste $J$ intorno di $\xbar$ tale che $a < f(x)$$\forall
x \in J \cap X \setminus\{\xbar\}$.
\end{exercise}
\end{exercise}
\begin{solution}
\begin{solution}
Si consideri $X =\{1+\frac{1}{n}\}\cup\{-1+\frac{1}{n}\}$.
Si consideri $g : X \to\RRbar$ tale che $g(x)= f(x)- a$. Poiché $g$ è una somma di
funzioni continue in $\xbar$, anch'essa è continua in $g$. Allora, poiché $g(\xbar) > 0$,
per il teorema della permanenza del segno, esiste un intorno $J$ di $\xbar$ tale per cui
$g(x) > 0$$\forall x \in J$, ossia tale per cui $f(x) > a$$\forall x \in J$, da cui la tesi.
\end{solution}
\end{solution}
\begin{exercise}
\begin{exercise}
Sia $f : X \to\RRbar$ continua in $\xbar$ e sia $a < f(\xbar)$.
Sia $X \subseteq\RRbar$ e sia $\xbar$ punto di accumulazione di $X$. Siano
Allora esiste $J$ intorno di $\xbar$ tale che $a < f(x)$$\forall
$f_1$, $f_2 : X \to\RRbar$. Si dimostri allora che:
x \in J$.
\begin{enumerate}[(i)]
\item se $f_1\tendsto{\xbar}+\infty$ e $f_2$ è limitata inferiormente
in un intorno $J$ di $\xbar$, allora $f_1(x)+ f_2(x)\tendsto{\xbar}+\infty$;
\item se $f_1\tendsto{\xbar}0$ e $f_2$ è limitata in un intorno $J$
di $\xbar$, allora $f_1 f_2(x)\tendsto{\xbar}0$;
\item se $f_1\tendsto{\xbar}+\infty$ è limitata inferiormente
da una costante positiva $c$ in un intorno $J$ di $\xbar$, allora
$f_1 f_2\tendsto{\xbar}+\infty$.
\end{enumerate}
\end{exercise}
\end{exercise}
\begin{exercise}
\begin{solution} Si dimostrano i tre risultati separatamente.
Sia $X \subseteq\RRbar$ e sia $\xbar$ punto di accumulazione di $X$,
$f_1$, $f_2 : X \to\RRbar$. Allora:
\begin{enumerate}[(i)]
\begin{enumerate}[(i)]
\item Se $f_1\tendsto{\xbar}+\infty$ e $f_2$ è limitata inferiormente
\item Sia $c$ la costante tale per cui $f_2(x)\geq c$$\forall x \in J \cap X$. Sia $I =[a, \infty]$ un intorno di $+\infty$. Se $c < 0$, poiché $f_1\tendsto{\xbar}+\infty$, esiste un intorno $J'$ tale per cui $f_1(J' \cap X \setminus\{\xbar\})\subseteq[a-c, \infty]\subseteq I$. Sia dunque $Z = J \cap J'$. Allora
in un intorno $J$ di $\xbar$, allora $f_1(x)+ f_2(x)\tendsto{\xbar}+\infty$.
$(f_1+ f_2)(x)= f_1(x)+ f_2(x)\geq a-c+c = a$$\forall x \in Z$, da cui si conclude che
\item Se $f_1\tendsto{\xbar}0$ e $f_2$ è limitata in un intorno
$(f_1+ f_2)(Z \cap X \setminus\{\xbar\})\subseteq I$. Se invece $c \geq0$, è sufficiente considerare
di $\xbar$, allora $f_1 f_2(x)\tendsto{\xbar}0$.
un intorno $J'$ di $\xbar$ tale per cui $f_1(J' \cap X \setminus\{\xbar\})\subseteq I$, da cui
\item Se $f_1\tendsto{\xbar}+\infty$ è limitata inferiormente
$(f_1+ f_2)(x)= f_1(x)+ f_2(x)\geq a+c \geq a$$\forall x \in Z \implies(f_1+f_2)(Z \cap X \setminus\{\xbar\})\subseteq I$, da cui la tesi.
da una costante positiva $m$ in un intorno $J$ di $\xbar$, allora
$f_1 f_2\tendsto{\xbar}+\infty$.
\item Poiché $f_2$ è limitata in $J$, esistono delle costanti finite $a$, $b \in\RR$ tali per cui $a \leq f_2(x)\leq b$$\forall x \in J$. Sia
\end{enumerate}
$I =[-\eps, \eps]$ un intorno di $0$, con $\eps > 0$. Si consideri $c :=\max\{\abs a, \abs b\}$.
\end{exercise}
Allora vale che $-c \leq f_2(x)\leq c$$\forall x \in J$. Poiché $f_1\tendsto{\xbar}0$, esiste
un intorno $J'$ di $\xbar$ tale per cui $f(J' \cap X \setminus\{\xbar\})\subseteq\left[-\frac{\eps}{c}, \frac{\eps}{c}\right]$. Si consideri ora $Z := J \cap J'$: vale allora che $\abs{(f_1 f_2)(x)}=\abs{f_1(x) f_2(x)}\leq c \frac{\eps}{c}=\eps$$\forall x \in Z \cap X \setminus\{\xbar\}$. Si conclude
dunque che $(f_1f_2)(Z \cap X \setminus\{\xbar\})\subseteq I$, da cui la tesi.
\item Sia $I =[a, \infty]$ un intorno di $+\infty$. Allora, poiché $f_1\tendsto{\xbar}+\infty$,
esiste un intorno $J'$ di $\xbar$ tale per cui $f_1(J' \cap X \setminus\{\xbar\})\subseteq[\abs a, \infty]\subseteq I$. Si
consideri dunque $Z := J \cap J'$: vale dunque che $(f_1f_2)(x)= f_1(x) f_2(x)\geq\abs{a}c \geq a$$\forall x \in Z \cap X \setminus\{\xbar\}$. Si conclude allora che $(f_1f_2)(Z \cap X \setminus\{\xbar\})\subseteq I$, da cui la tesi.
\end{enumerate}
\end{solution}
\begin{exercise}
\begin{exercise}
Sia $f: \RR\to\RR$ tale che:
Sia $f: \RR\to\RR$ tale che:
@ -325,6 +371,26 @@
\vskip 0.05in
\vskip 0.05in
Mostrare che $f$ è continua, che $f'(0)=1$ e che $f'$ non è continua in zero.
Si mostri che $f$ è continua ovunque e che $D_+ f(0)=1$.
\end{exercise}
\end{exercise}
\begin{solution}
Poiché somma di funzioni elementari, $f$ è continua in $(0, \infty)$. Analogamente è continua in
$(-\infty, 0)$ dacché è costante in tale intervallo. Affinché allora $f$ sia continua ovunque è
sufficiente che si dimostri che è continua anche in $0$. Dal momento che $0$ è un punto di accumulazione
sia destro che sinistro di $\RR$, questo equivale a mostrare che il limite destro e sinistro di $f$